Marketing consultant: Last year I predicted that LRG's latest advertising campaign would be unpopular with customers ...

smilde11 on October 24, 2018

PT 73, S2, Q15

Can you explain why A would not be correct? Thank you!

Reply
Create a free account to read and take part in forum discussions.

Already have an account? log in

Mehran on October 25, 2018

Hi @smilde11, thanks for your post. Let's first assess the stimulus. This one presents an argument; the conclusion is"the advertising campaign was ill conceived." What premises are provided in support of this claim? The marketing consultant says (1) I predicted that the campaign would be unpopular with customers and ineffective in promoting new products; (2) LRG ignored me and took a competing consultant's advice; and (3) this season's sales figures show that sales are down and LRG's new products are selling especially poorly."

This is a cause & effect argument. The effect is "sales are down" and "new products are selling especially poorly." The cause, according to the marketing consultant, is "the advertising campaign was ill conceived." There are three ways you can weaken a cause & effect argument: (1) show an alternative cause, (2) show that the effect exists without the expected cause, or (3) show that the cause exists without the expected effect.

The question stem is an Errors in Reasoning question ("reasoning is most vulnerable to criticism on the grounds that"). The trouble with this argument is that it ignores other possible causes for the described effect.

Answer choice (B) articulates this flaw and identifies an alternative cause ("economic factors unrelated to the advertising campaign") for the effect ("sales are down" and "new products are selling especially poorly"). This is why it is the correct answer.

Answer choice (A) is incorrect because it does not address THIS conclusion. Look very closely at the specific conclusion presented to you in each stimulus you read. Here, the conclusion is that the advertising campaign the company DID go with was ill conceived. That's got nothing to do with the competitor's advertising campaign. Notice also the premises given to support the conclusion: the key premise is about sales being down and new products selling poorly. We need to focus on the link between these given premises and the given conclusion to weaken THAT argument, not argue with the competitor marketing consultant that his campaign would have been even worse.

Hope that helps. Let us know if you have any additional questions. Again: really focus on the scope of the specific argument/conclusion in front of you. If you get stuck between two answer choices, go back and re-read the stimulus. It will help unstick you.